2
$\begingroup$

Let $A\in\mathbb{R}^{m\times d}$ matrix with iid standard normal entries, and $m\geqslant d$, and define $S=A^T A$.

I want to have a tight upper bound for $\sum_{k=1}^d \lambda_k^2$, where $\lambda_1,\dots,\lambda_d$ are the eigenvalues of $S$.

What I tried:

  • We know that (see e.g. Corollary 5.35 in Vershynin's notes), for $A\in\mathbb{R}^{m\times d}$, for any $t\geqslant 0$, with probability at least $1-2\exp(-\Omega(t^2))$, it holds: $$ \sqrt{m}-\sqrt{d}-t \leqslant \sigma_{min}(A)\leqslant \sigma_{max}(A)\leqslant \sqrt{m}+\sqrt{d}+t. $$ Simply ignoring $\sqrt{d},t$ terms (say I am in the regime $m\gg d,t$), this yields $\lambda_i(A)<m^2$, and thus, the sum above is upper bounded by $m^2d$.
  • We also have the following: $$ \sum_{k=1}^d (\lambda_k - m) = \sum_{i =1}^m \sum_{j=1}^d (A_{ij}^2-1), $$ which is sum of sub-exponential random variables, and thus, by a Bernstein-type bound, $\sum_{k=1}^d \lambda_k \leqslant md+\omega(\sqrt{md})$, for some function $\omega(\sqrt{md})$ growing faster than $\sqrt{md}$.
  • The sum above is simply the trace of $S^2=A^TAA^TA$.

I'm new to random matrix business, so any help is greatly appreciated.

$\endgroup$

3 Answers 3

1
$\begingroup$

I will assume $m=\alpha_d d$ with $\alpha_d\to \alpha \in [1,\infty)$ independent of $d$. The case $\alpha\to\infty$ is actually easier.

Define $Z=d^{-1} m^{-2} \sum_{i=1}^d \lambda_i^2$. Then $Z$ converges a.s. to $\int x^2 d\mu_\alpha(x)$ where $\mu_\alpha$ is the Pastur-Marchenko distribution of parameter $\lambda=1/\alpha$, see https://en.wikipedia.org/wiki/Marchenko%E2%80%93Pastur_distribution

$\endgroup$
5
  • $\begingroup$ Ofer, thanks for the answer. Sorry, if it is a trivial question: How to handle the case $\alpha\to+\infty$ (more precisely, when $d\to+\infty$ and $d=o(m)$)? $\endgroup$
    – hookah
    Jun 17, 2019 at 20:13
  • $\begingroup$ And also why the normalization is by $d^{-1}m^{-2}$? I understand that MP law requires $\frac1m$ normalization, and you get an extra for $d$, but why $d^2$? $\endgroup$
    – hookah
    Jun 17, 2019 at 20:19
  • $\begingroup$ That is the limiting case, in which the limit empirical measure is a dirac at 1.... $\endgroup$ Jun 17, 2019 at 20:19
  • $\begingroup$ and the normalization is as in the wikipedia page I quoted. You get $1/d$ from the empirical measure and $1/m^2$ from the normalization of the entries (you are taking the square of the MP matrix...) $\endgroup$ Jun 17, 2019 at 20:22
  • 1
    $\begingroup$ @oferzeitouni Possible typo: $m=\alpha_d s$ ==> $m=\alpha_d d$. $\endgroup$
    – dohmatob
    Jun 29, 2021 at 11:48
0
$\begingroup$

The problem can be solved using only elementary arguments (i.e without RMT).

Claim. In the limit $d,m \to \infty$ such that $m/d \to \rho \in (0,\infty)$, it holds that $$ m^{-1}d^{-2}\sum_{i}\lambda_i(S)^2 \overset{a.s}{\to} 1+\rho. $$

Indeed, one may write $$ \sum_{i}\lambda_i(S)^2 = \sum_{i,j=1}^m s_{i,j}^2 = \sum_{i,j} (a_i^\top a_j)^2 = \sum_{i=1}^m(\|a_i\|^4 + \sum_{j\ne i}^m (a_i^\top a_j)^2). \tag{1} $$ Now, by the Law of Large numbers, it's clear that $d^{-2}\|a_i\|^4 = (d^{-1}\|a_i\|^2)^2 \overset{a.s}{\to} 1^2 = 1$ for all $i \in [m]$. On the other hand, if $i \ne j$, then $d^{-2}(a_i^\top a_j)^2$ has a beta distribution with parameters $\alpha=1$ and $\beta=d-2$ (see this post https://mathoverflow.net/a/227156/78539), and expected value $\alpha / (\alpha + \beta) = 1/(d-1)$. Combining with (1) via another application of LLNs (to handle the second term) then completes the proof after noting that $(m-1) / (d-1) \to \rho$.

Question. Comparing with the accepted answer, is true that $\langle \lambda^2\rangle_{MP(1/\rho)} = 1 + \rho$ ?

$\endgroup$
0
$\begingroup$

The joint probability distribution of the eigenvalues of $S$ is proportional to $$ \rho(S)=e^{-{\rm Tr}(S)}\det(S)^{a/2}|\Delta(S)|,$$ where $a=m-d-1$ and $\Delta(S)$ is the Vandermonde. The average value of any symmetric function $f$ of the eigenvalues can be computed exactly by writing $f$ as a linear combination of Zonal polynomials, $f(S)=\sum_\lambda c_\lambda Z_\lambda(S)$, and then using the explicit result of the Selberg-like integral $$ \int_0^\infty Z_\lambda(S)\rho(S)dS,$$ which can be found e.g. in The Importance of the Selberg Integral, by P. Forrester and O. Warnaar.

$\endgroup$

Your Answer

By clicking “Post Your Answer”, you agree to our terms of service and acknowledge you have read our privacy policy.

Not the answer you're looking for? Browse other questions tagged or ask your own question.